Recast a given vector field F in cylindrical coordinates

Click For Summary
The discussion focuses on converting the vector field F(x,y,z) = xzi into cylindrical coordinates, resulting in F(r,θ,z) = rcos(θ)z. Participants discuss the calculation of the curl in both Cartesian and cylindrical coordinates, noting discrepancies in their results. The correct transformation of the unit vector i into cylindrical coordinates is emphasized, which is crucial for obtaining consistent results. After identifying and correcting mistakes, one participant confirms that their answers for the curl in both coordinate systems agree. The conversation highlights the importance of careful coordinate transformations in vector calculus.
Bestphysics112
Messages
24
Reaction score
2

Homework Statement


F(x,y,z) = xzi

Homework Equations


N/A

The Attempt at a Solution


I just said that x = rcos(θ) so F(r,θ,z) = rcos(θ)z. Is this correct? Beaucse I am also asked to find curl of F in Cartesian coordinates and compare to curl of F in cylindrical coordinates. For Curl of F in cylindrical coordinates I obtained rcos(θ)eθ+sin(θ)zez. This doesn't look anything like the curl found in Cartesian coordinates. Where am i going wrong?
 
Physics news on Phys.org
The unit vector ## \hat{i} ## also needs to be put in cylindrical coordinates. ## \hat{i}=cos(\theta) \hat{a}_r -sin(\theta) \hat{a}_{\theta} ## if I computed it correctly. Once you do that, you should get agreement when you do the curl operation in cylindrical coordinates. ## \\ ## Editing: Yes, I computed it, and got agreement. I'd be happy to check the answer for you that you get.
 
Last edited:
  • Like
Likes Bestphysics112
Charles Link said:
The unit vector ## \hat{i} ## also needs to be put in cylindrical coordinates. ## \hat{i}=cos(\theta) \hat{a}_r -sin(\theta) \hat{a}_{\theta} ## if I computed it correctly. Once you do that, you should get agreement when you do the curl operation in cylindrical coordinates. ## \\ ## Editing: Yes, I computed it, and got agreement. I'd be happy to check the answer for you that you get.
Hello! Sorry I just saw this reply. As an answer, I got (rcos(θ)sin(θ)er +rcos2(θ)eθ +(2cos(θ)sin(θ)z+ 2sin(θ)cos(θ)z)ez

Is this answer correct?

Edit- Yes! I figured out my mistake and I got an equivalent answer. Thank you for the help Charles
 
Last edited:
  • Like
Likes Charles Link
Bestphysics112 said:
Hello! Sorry I just saw this reply. As an answer, I got (rcos(θ)sin(θ)er +rcos2(θ)eθ +(2cos(θ)sin(θ)z+ 2sin(θ)cos(θ)z)ez

Is this answer correct?

Edit- Yes! I figured out my mistake and I got an equivalent answer. Thank you for the help Charles
Please check your ## e_z ## term. The first term gets a minus sign so that the ## e_z ## term is zero. Also what did you get in Cartesian coordinates for the curl? The two should agree and I think they do if the ## e_z ## term is zero.
 
Charles Link said:
Please check your ## e_z ## term. The first term gets a minus sign so that the ## e_z ## term is zero. Also what did you get in Cartesian coordinates for the curl? The two should agree and I think they do if the ## e_z ## term is zero.
I don't have my homework with me, but I forgot to edit my post with the answer I got. The ez terms canceled
 
  • Like
Likes Charles Link
Question: A clock's minute hand has length 4 and its hour hand has length 3. What is the distance between the tips at the moment when it is increasing most rapidly?(Putnam Exam Question) Answer: Making assumption that both the hands moves at constant angular velocities, the answer is ## \sqrt{7} .## But don't you think this assumption is somewhat doubtful and wrong?

Similar threads

Replies
2
Views
2K
  • · Replies 4 ·
Replies
4
Views
2K
Replies
2
Views
1K
  • · Replies 9 ·
Replies
9
Views
2K
  • · Replies 10 ·
Replies
10
Views
1K
Replies
33
Views
4K
  • · Replies 17 ·
Replies
17
Views
4K
  • · Replies 6 ·
Replies
6
Views
3K
Replies
4
Views
2K
  • · Replies 2 ·
Replies
2
Views
2K